Re: I have questions
« Reply #40 on: October 15, 2019, 02:48:23 PM »
You are the only person in the world who claims this.

My previous message proved that EVERYONE at ESA and NASA knows and accepts the fact that for the LISA Space Antenna, the interferometer will record BOTH the Coriolis effect and the Sagnac effect.



In this work, we estimate the effects due to the Sagnac phase by taking the realistic model for LISA orbital motion.

This work is organized as follows: in section 2, we make an estimate of Sagnac phase
for individual laser beams of LISA by taking realistic orbital motion. Here we show that, in general, the residual laser noise because of Sagnac phase is much larger than earlier estimates.

For the LISA geometry, R⊙/L is of the order 30 and the orbital contribution to the Sagnac phase is larger by this factor.

The computations carried out by Dr. R.K. Nayak (over ten papers published on the subject) and Dr. J.Y. Vinet (Member of the LISA International Science Team), and published by prestigious scientific journals and by ESA, show that the orbital Sagnac is 30 times greater than the rotational Sagnac for LISA.


The same phenomenon is at work for the MGX and RLGs.

One has an interferometer which is rotating on the surface of a sphere: it will be subjected to both the Coriolis effect and to the Sagnac effect.


You still do not seem to understand what is going on.

Nobody is contending the existence of ether. The relativists have begun to fully accept this fact; they have to, since the GPS satellites do not record the orbital Sagnac effect.

If you agree that Michelson published the SAGNAC EFFECT formula, then the RE win hands down.

Can you understand that the Sagnac effect proves the ROTATION of the interferometer itself?


Here is a second proof, using Stokes' theorem:



https://www.theflatearthsociety.org/forum/index.php?topic=30499.msg2208660#msg2208660

According to Stokes' rule an integration of angular velocity Ω over an area A is substituted by an integration of tangential component of translational velocity v along the closed line of length L limiting the given area:

https://www.theflatearthsociety.org/forum/index.php?topic=30499.msg2023979#msg2023979

In the case where the interferometer will be located away from the center of rotation (MGX/RLGs), there will be a factor of proportionality: R/L, where R = radius of the Earth.



This factor of proportionality was proven, for the first time, for the LISA Space Antenna, as shown above.




Thus we can see the huge discrepancy between the CORIOLIS EFFECT formula and the SAGNAC EFFECT formula for the MGX/RLGs (Michelson-Gale experiment/Ring Laser Gyroscopes).

There is NO velocity term for the CORIOLIS EFFECT formula: 4Aω/c2.

Therefore, if the interferometer itself is being rotated, and the RADIUS of rotation is R, then the SAGNAC EFFECT will be proportional to the velocity, V = R x ω.


A = L x l (long side multiplied by the short side)

Then, 4Aω = 4Vωl, where Vω = L x ω.

That is, the CORIOLIS EFFECT formula is equivalent to a SAGNAC EFFECT formula where the RADIUS of rotation now becomes L.

Since the RADIUS of rotation in the MGX/RLGs is actually the radius of the Earth, it becomes obvious that the SAGNAC EFFECT formula will be greater than the CORIOLIS EFFECT formula by at least a factor of R/l (interferometer in the shape of a rectangle), exactly the term found by CalTech and ESA for the LISA Space Antenna.

*

Offline TomInAustin

  • *
  • Posts: 1367
  • Round Duh
    • View Profile
Re: I have questions
« Reply #41 on: October 15, 2019, 02:59:59 PM »
You are the only person in the world who claims this.

My previous message proved that EVERYONE at ESA and NASA knows and accepts the fact that for the LISA Space Antenna, the interferometer will record BOTH the Coriolis effect and the Sagnac effect.



In this work, we estimate the effects due to the Sagnac phase by taking the realistic model for LISA orbital motion.

This work is organized as follows: in section 2, we make an estimate of Sagnac phase
for individual laser beams of LISA by taking realistic orbital motion. Here we show that, in general, the residual laser noise because of Sagnac phase is much larger than earlier estimates.

For the LISA geometry, R⊙/L is of the order 30 and the orbital contribution to the Sagnac phase is larger by this factor.

The computations carried out by Dr. R.K. Nayak (over ten papers published on the subject) and Dr. J.Y. Vinet (Member of the LISA International Science Team), and published by prestigious scientific journals and by ESA, show that the orbital Sagnac is 30 times greater than the rotational Sagnac for LISA.


The same phenomenon is at work for the MGX and RLGs.

One has an interferometer which is rotating on the surface of a sphere: it will be subjected to both the Coriolis effect and to the Sagnac effect.


You still do not seem to understand what is going on.

Nobody is contending the existence of ether. The relativists have begun to fully accept this fact; they have to, since the GPS satellites do not record the orbital Sagnac effect.

If you agree that Michelson published the SAGNAC EFFECT formula, then the RE win hands down.

Can you understand that the Sagnac effect proves the ROTATION of the interferometer itself?


Here is a second proof, using Stokes' theorem:



https://www.theflatearthsociety.org/forum/index.php?topic=30499.msg2208660#msg2208660

According to Stokes' rule an integration of angular velocity Ω over an area A is substituted by an integration of tangential component of translational velocity v along the closed line of length L limiting the given area:

https://www.theflatearthsociety.org/forum/index.php?topic=30499.msg2023979#msg2023979

In the case where the interferometer will be located away from the center of rotation (MGX/RLGs), there will be a factor of proportionality: R/L, where R = radius of the Earth.



This factor of proportionality was proven, for the first time, for the LISA Space Antenna, as shown above.




Thus we can see the huge discrepancy between the CORIOLIS EFFECT formula and the SAGNAC EFFECT formula for the MGX/RLGs (Michelson-Gale experiment/Ring Laser Gyroscopes).

There is NO velocity term for the CORIOLIS EFFECT formula: 4Aω/c2.

Therefore, if the interferometer itself is being rotated, and the RADIUS of rotation is R, then the SAGNAC EFFECT will be proportional to the velocity, V = R x ω.


A = L x l (long side multiplied by the short side)

Then, 4Aω = 4Vωl, where Vω = L x ω.

That is, the CORIOLIS EFFECT formula is equivalent to a SAGNAC EFFECT formula where the RADIUS of rotation now becomes L.

Since the RADIUS of rotation in the MGX/RLGs is actually the radius of the Earth, it becomes obvious that the SAGNAC EFFECT formula will be greater than the CORIOLIS EFFECT formula by at least a factor of R/l (interferometer in the shape of a rectangle), exactly the term found by CalTech and ESA for the LISA Space Antenna.

I am confused.  Youa re a flat earther but you mention satliltes as proof of your theorys?  The LISA Space Antenna operates outside of Earth's sphere of influence.  If you accept that you have to accept that the earth is not flat.
Do you have a citation for this sweeping generalisation?

Re: I have questions
« Reply #42 on: October 15, 2019, 03:11:57 PM »
The LISA Space Antenna operates outside of Earth's sphere of influence.

The LISA Space Antenna is supposed to operate outside of Earth's sphere of influence.

I have just stated that fact on page 2 of this thread.

Given the huge cost of the entire project, the best experts in the field (CalTech, ESA) were called upon to provide the necessary theoretical calculations for the total phase shift of the interferometer. To everyone's surprise, and for the first time since Sagnac and Michelson and Gale, it was found that the ORBITAL SAGNAC EFFECT is much greater than the CORIOLIS EFFECT.

The factor of proportionality is R/L (R = radius of rotation, L = length of the side of the interferometer).



GPS satellites orbit at a much lower altitude using the Biefeld-Brown effect.

*

Offline TomInAustin

  • *
  • Posts: 1367
  • Round Duh
    • View Profile
Re: I have questions
« Reply #43 on: October 15, 2019, 03:17:51 PM »
The LISA Space Antenna operates outside of Earth's sphere of influence.

The LISA Space Antenna is supposed to operate outside of Earth's sphere of influence.


So it either does or it doesn't, which is it?

If it does it proves round earth, if it doesn't you can't use its findings as evidence.
« Last Edit: October 15, 2019, 03:19:32 PM by TomInAustin »
Do you have a citation for this sweeping generalisation?

Re: I have questions
« Reply #44 on: October 15, 2019, 03:46:01 PM »
You haven't done your homework on the subject.

ESA claims that the LISA Pathfinder mission was in orbit (2015-2017).

Why didn't the LISA Pathfinder satellite register/record the orbital and the galactic Sagnac effects?

*

Offline TomInAustin

  • *
  • Posts: 1367
  • Round Duh
    • View Profile
Re: I have questions
« Reply #45 on: October 15, 2019, 03:52:26 PM »
You haven't done your homework on the subject.
ESA claims that the LISA Pathfinder mission was in orbit (2015-2017).

Stop trying to deflect. 

Does it exist or not? 

[ ] Yes
[ ] No


Do you have a citation for this sweeping generalisation?

Re: I have questions
« Reply #46 on: October 15, 2019, 04:07:31 PM »
If you agree that Michelson published the SAGNAC EFFECT formula, then the RE win hands down.
Can you understand that the Sagnac effect proves the ROTATION of the interferometer itself?


RE win hands down?
What the shape of the earth has to do with this?
You meant to say : HC win hands down?
No HC don't win hands down, GC win hands down!

WIKI QUOTE :
Because the Earth spins, Earth-bound observers need to account for the Coriolis force to correctly analyze the motion of objects. The Earth completes one rotation per day, so for motions of everyday objects the Coriolis force is usually quite small compared to other forces; its effects generally become noticeable only for motions occurring over large distances and long periods of time, such as large-scale movement of air in the atmosphere or water in the ocean.

Be that as it may, the Sagnac effect is seen in many places, since it is an established fact that two beams of light heading in different directions are going to have a discrepancy in their speed. This is shown by the Sagnac correction that is built into the GPS system to make it work. EM waves going east-to-west go faster than EM waves going west-to-east, but  the  distance  between the GPS satellite remains the same, and thus the GPS computers have to be adjusted by the Sagnac results to account for the anisotropy of the EM waves.

Moreover, there does not exist a peer-reviewed paper that has ever disputed the results of 1925 MGX, much less disprove them.  In fact, if anything, the results of that  experiment are virtually kept silent in modern academia. I believe there is a good reason for this silence, for if it became known that the same principle of ether-drift was used in two different experiments (MMX and MGX) measuring two different things (revolution v. rotation) and the former showed no drift but the latter showed a full drift, the physics world  would  be  in  a  total  conundrum. It would virtually prove that the Earth was  motionless in space and that the universe rotated around it.  The option that the Earth could be rotating but not revolving around the sun would not be possible, because in that scenario the Earth could not show the seasons, only day and night.

So, the heliocentrists spend all their time trying to show that MMX-type experiments are null and then try to claim this means light is constant and the Earth moves. This takes our eyes off the more important results in the 1925 MGX experiment in which the ether exists and the speed of light is not constant (besides the option that the alternative interpretation of MMX is that the Earth doesn’t move and light is not constant).

Since MGX measured 98% of the ether drift expected for a 0.46km/sec rotation, we have all the evidence we need, not only for a daily rotation but for the existence of the ether, since MGX could only measure ether, since the experiment was done in a vacuum, not in a gaseous medium.

We have to ask this question : Had coriolis force had any effect on MGPX what kind of effect would have it been?
Would coriolis force have impeded light beams or would have it deflected them?
If we suppose that coriolis force had had an impeding effect, how significant would have been such an effect on light beams, and in which direction?
Impeding effect on light beams that travel in one direction would be canceled out by acceleration of light beams that travel in an opposite direction.
On the other hand, deflecting effect would cause light beams to miss the target (the receiver) so that correct reading would be impossible (how could light waves travel in a straight line, anyway, if coriolis force could affect light?)...
All in all, bringing coriolis force in the context of MGPX is in my opinion a complete idiocy...

Re: I have questions
« Reply #47 on: October 15, 2019, 04:23:54 PM »
WIKI QUOTE :
Because the Earth spins, Earth-bound observers need to account for the Coriolis force to correctly analyze the motion of objects. The Earth completes one rotation per day, so for motions of everyday objects the Coriolis force is usually quite small compared to other forces; its effects generally become noticeable only for motions occurring over large distances and long periods of time, such as large-scale movement of air in the atmosphere or water in the ocean.


For light beams, the effect is readily and immediately measurable.

Here is the CORIOLIS EFFECT formula:


https://www.ias.ac.in/article/fulltext/pram/087/05/0071

Spinning Earth and its Coriolis effect on the circuital light beams

The final formula is this:

dt = 4ωA/c2


The SAGNAC EFFECT, by contrast, does not feature an AREA at all.




NO ENCLOSED AREA APPEARS IN THIS EXPRESSION.


This is shown by the Sagnac correction that is built into the GPS system to make it work. EM waves going east-to-west go faster than EM waves going west-to-east, but  the  distance  between the GPS satellite remains the same, and thus the GPS computers have to be adjusted by the Sagnac results to account for the anisotropy of the EM waves.

What formula did you use for the correction?

This one?

dt = 4ωA/c2

That is the Coriolis effect formula.

None other than Neil Ashby, the high priest of relativity theory applied to GPS satellites agrees.

In a rare admission, even N. Ashby states that the Coriolis force is responsible for the term commonly used in GPS technology for the Sagnac effect:





We have to ask this question : Had coriolis force had any effect on MGPX what kind of effect would have it been?

I already answered that question from the very start: the effect is physical, a slight deflection of the light beam, which still will reach the target, the deflection is in the middle of the trajectory.

Here is the diagram drawn by Dr. Ludwik Silberstein:


*

Offline Tom Bishop

  • Zetetic Council Member
  • **
  • Posts: 10638
  • Flat Earth Believer
    • View Profile
Re: I have questions
« Reply #48 on: October 15, 2019, 06:23:52 PM »
Quote
Your statements are very eloquent and would be applicable to our discussion here if and only if you can disprove the RE's main contention point: the SAGNAC EFFECT formula proves rotation.

Lets assume that the Michelson-Gale equation is correct or that the formula is analogous to a Sagnac equation:

How can this Sagnac Effect from the device prove rotation of earth's surface when we know that there are other variables which are not caused by the rotation of the earth that can cause the results to change drastically?

We saw from the inconsistency that the rotation of the earth in the Michelson-Gale is dominated by another effect which is not the rotation of the earth. This proves that the Sagnac Effect doesn't necessarily measure rotation of the earth. If it detects something then we can say that it might or might not be rotation of the surface. Whether it is the Sagnac formula or not, and despite nitpicking of formulas, the inconsistency and dominance of other effects in the experiment shows that the mechanism and associated formula is not reliable to prove rotation.
« Last Edit: October 15, 2019, 06:41:47 PM by Tom Bishop »

Re: I have questions
« Reply #49 on: October 15, 2019, 08:52:23 PM »
How can this Sagnac Effect from the device prove rotation of earth's surface when we know that there are other variables which are not caused by the rotation of the earth that can cause the results to change drastically?

We saw from the inconsistency that the rotation of the earth in the Michelson-Gale is dominated by another effect which is not the rotation of the earth. This proves that the Sagnac Effect doesn't necessarily measure rotation of the earth.


You are not describing the Sagnac effect, but the Coriolis effect.

The inconsistency is due to the fact that the ether drift is latitude dependent and sinusoidal in nature.



However, the statistics work in the RE's favor:



They will simply use the formula derived by Michelson and claim victory:



The figures for the area of the path, latitude (41deg. 46'), wavelength of the light, speed of light, and the expected fringe shifts are well known.

Expected fringe shift: 0.2364

Measured fringe shift: 0.230 +/- 0.005

Then, the angular velocity of the Earth can be easily computed.


The RE have the precise formula, the other participants in the discussion have NOTHING.


The SAGNAC EFFECT will always prove the rotation of the interferometer itself and Michelson claimed that the formula derived by him is the SAGNAC EFFECT formula. It is this claim that you have to disprove if you want to explain the experiment.

As for the RLGs,  both terrestrial and those used in airplanes, using seismic waves and/or Earth's line fluctuations: these can be readily explained by the RE.

The same formula is used for RLGs: since the RE say that this formula is the SAGNAC EFFECT equation, they now have proven ROTATION as well.

You can only counteract these claims with the TRUE Sagnac formula itself.




Re: I have questions
« Reply #50 on: October 16, 2019, 10:45:32 AM »
However, the statistics work in the RE's favor:

They will simply use the formula derived by Michelson and claim victory:

The figures for the area of the path, latitude (41deg. 46'), wavelength of the light, speed of light, and the expected fringe shifts are well known.
Expected fringe shift: 0.2364
Measured fringe shift: 0.230 +/- 0.005
Then, the angular velocity of the Earth can be easily computed.
The RE have the precise formula, the other participants in the discussion have NOTHING.
The other participants in the discussion = Flat Earthers (exclusively)

So, since you believe that the earth is flat, and MGPX presumes spherical shape of the earth, then you have to find the way how to discredit MGPX, so that you can keep claiming that the earth is flat. Jesus Christ, even Aristotles (2500 years ago) knew that the earth was spherically shaped...

The SAGNAC EFFECT will always prove the rotation of the interferometer itself and Michelson claimed that the formula derived by him is the SAGNAC EFFECT formula. It is this claim that you have to disprove if you want to explain the experiment.

The SAGNAC EFFECT will not always prove the rotation of the interferometer itself!!!

The SAGNAC EFFECT proves rotation of aether, once again just for you :

Moreover, there does not exist a peer-reviewed paper that has ever disputed the results of 1925 MGX, much less disprove them.  In fact, if anything, the results of that  experiment are virtually kept silent in modern academia. I believe there is a good reason for this silence, for if it became known that the same principle of ether-drift was used in two different experiments (MMX and MGX) measuring two different things (revolution v. rotation) and the former showed no drift but the latter showed a full drift, the physics world  would  be  in  a  total  conundrum. It would virtually prove that the Earth was  motionless in space and that the universe rotated around it.  The option that the Earth could be rotating but not revolving around the sun would not be possible, because in that scenario the Earth could not show the seasons, only day and night.

So, the heliocentrists spend all their time trying to show that MMX-type experiments are null and then try to claim this means light is constant and the Earth moves. This takes our eyes off the more important results in the 1925 MGX experiment in which the ether exists and the speed of light is not constant (besides the option that the alternative interpretation of MMX is that the Earth doesn’t move and light is not constant).

So, Sandokhan spends all his time trying to show that MGPX is not valid so that he can continue to claim that the earth is flat! OMG

For those who don't understand how and why MGPX presumes spherical shape of the earth :

Relative motion of earth and Aether (Sandokhan, pay attention to the word in red) : http://www.conspiracyoflight.com/Michelson-Gale/Michelson_1904.pdf
Quote from an article above :

ASSERTION NO 1 :
The system of interference-fringes produced by the superposition of the two pencils-one of which has traversed the circuit clockwise, and the other counterclockwise-would be shifted through seven-tenths of the distance between the fringes, in the direction corresponding to a retardation of the clockwise pencil, if the experiment were tried in the Northern hemisphere.

So the experiment has been carried out in the Northern hemisphere :
Now the quote from this article : http://ether-wind.narod.ru/Michelson_Gale_1925/Michelson_Gale_1925.pdf

ASSERTION NO 2 :
Experimental  Test  of  Theory.— Air was exhausted from a twelve-inch pipe line laid on the surface of the ground in the form of  a  rectangle  2010× 1113  feet.  Light  from  a carbon arc was divided at one comer by a thinly coated mirror into direct and reflected beams,  which were reflected  around  the  rectangle by mirrors  at  the  comers.  The  two beams returning to  the original mirror produced interference  fringes.  The beam trav­ersing  the  rectangle in  a  counter-clockwise  direction  was  retarded.  The  observed  dis­placement of the fringes was found to be 0.230± .005, agreeing with the computed value0.236± .002 within the limits of experimental error.

Now, one little technical question for Sandokhan :
How come that the beam traversing the rectangle in a counter-clockwise direction was retarded (ASSERTION NO 2) given what is stated in ASSERTION NO 1?

Re: I have questions
« Reply #51 on: October 16, 2019, 11:04:15 AM »
The SAGNAC EFFECT will not always prove the rotation of the interferometer itself!!!

The SAGNAC EFFECT proves rotation of aether


The hypothesis leading to the Sagnac effect REQUIRES the existence of ether.

This is accepted by the relativists.

The hallmark of the SAGNAC EFFECT is exactly the precise proof that the interferometer itself (turntable, satellite, planet) is ROTATING.

You must study this field of physics a little deeper.


Let us remember what happened to the GE/FE on youtube when B. Knodel tried to prove that the Earth is at rest using ring laser gyroscopes.

The RE demolished each and everyone of their arguments, across dozens of youtube videos, having gained the upper hand in each and every discussion.


Why?

Because the GE/FE had nothing at their disposal, no formula at all, while the RE were triumphant.

Until now.

How come that the beam traversing the rectangle in a counter-clockwise direction was retarded (ASSERTION NO 2) given what is stated in ASSERTION NO 1?

You extracted quotes from papers published 21 years apart.


Now, here is the best formula in all of physics, the TRUE Sagnac effect formula:





Point A is located at the detector
Point B is in the bottom right corner
Point C is in the upper right corner
Point D is in the upper left corner

l1 is the upper arm.
l2 is the lower arm.

Here is the most important part of the derivation of the full/global Sagnac effect for an interferometer located away from the center of rotation.

A > B > C > D > A is a continuous counterclockwise path, a negative sign -

A > D > C > B > A is a continuous clockwise path, a positive sign +

The Sagnac phase difference for the clockwise path has a positive sign.

The Sagnac phase difference for the counterclockwise has a negative sign.


Sagnac phase components for the A > D > C > B > A path (clockwise path):

l1/(c - v1)

-l2/(c + v2)

Sagnac phase components for the A > B > C > D > A path (counterclockwise path):

l2/(c - v2)

-l1/(c + v1)


For the single continuous clockwise path we add the components:

l1/(c - v1) - l2/(c + v2)

For the single continuous counterclockwise path we add the components:

l2/(c - v2) - l1/(c + v1)


The net phase difference will be (let us remember that the counterclockwise phase difference has a negative sign attached to it, that is why the substraction of the phase differences becomes an addition):

{l1/(c - v1) - l2/(c + v2)} - (-){l2/(c - v2) - l1/(c + v1)} = {l1/(c - v1) - l2/(c + v2)} + {l2/(c - v2) - l1/(c + v1)}

Rearranging terms:

l1/(c - v1) - l1/(c + v1) + {l2/(c - v2) - l2/(c + v2)} =

2(v1l1 + v2l2)/c2


Re: I have questions
« Reply #52 on: October 16, 2019, 03:34:43 PM »
How come that the beam traversing the rectangle in a counter-clockwise direction was retarded (ASSERTION NO 2) given what is stated in ASSERTION NO 1?

You extracted quotes from papers published 21 years apart.

So what? We are discussing the results of an experiment which was conducted 94 years ago, and still (94 years later) there does not exist a peer-reviewed paper that has ever disputed the results of 1925 MGX, much less disprove them.

Thanks for your exhaustive math procedure intended to answer my question, but i would appreciate if you could answer that question in layman terms (without using mathematical formulas) : how come that Michelson 21 years ahead (in 1904) claims something which is in direct contradiction with the result of his experiment conducted in 1925?

Once again :

ASSERTION NO 1 :
The system of interference-fringes produced by the superposition of the two pencils-one of which has traversed the circuit clockwise, and the other counterclockwise-would be shifted through seven-tenths of the distance between the fringes, in the direction corresponding to a retardation of the clockwise pencil, if the experiment were tried in the Northern hemisphere.

ASSERTION NO 2 :
Experimental  Test  of  Theory.— Air was exhausted from a twelve-inch pipe line laid on the surface of the ground in the form of  a  rectangle  2010× 1113  feet.  Light  from  a carbon arc was divided at one comer by a thinly coated mirror into direct and reflected beams,  which were reflected  around  the  rectangle by mirrors  at  the  comers.  The  two beams returning to  the original mirror produced interference  fringes.  The beam trav­ersing  the  rectangle in  a  counter-clockwise  direction  was  retarded.  The  observed  dis­placement of the fringes was found to be 0.230± .005, agreeing with the computed value0.236± .002 within the limits of experimental error.

Let's put it more directly : why the beam traversing the rectangle in a counter-clockwise direction was retarded?
Or to be even more precise : why the beam traversing the rectangle in a counter-clockwise direction (not in a clockwise direction, as Michelson stated in his earlier paper (from 1904.)) would have to be retarded if the experiment were tried in the Northern hemisphere?
Care to answer it in layman terms (for our broader audience)...I know the correct answer to this question, however i guess many of our viewers would have difficulties with answering that question relying solely on their own knowledge...so...let's try to be educative...
« Last Edit: October 16, 2019, 03:38:15 PM by cikljamas »

Re: I have questions
« Reply #53 on: October 16, 2019, 04:03:55 PM »
Michelson was very careful not to reveal what he meant by "the light beam was retarded".

Because there are two possibilities.

1. The beams suffered a slight lateral deviation.

2. The velocities of the beams underwent a lag.

BOTH should have occurred during the MGX.

But Michelson only mentions one type of retardation.

Since the formula he used is proportional to the area of the interferometer, it is obvious that he registered the first version of the "retardation" of the beams.

The real reason for the retardation of the counterclock-wise beam can be found here, a paper posted earlier:

http://cdn.intechopen.com/pdfs/39778/InTech-Gps_and_the_one_way_speed_of_light.pdf

"Light travels faster westward than eastward relative to the surface of the Earth. Specifically
the one-way measurement of light speed using GPS data in (6) clearly indicates that a signal
sent eastward travels at speed c minus the rotational speed of the Earth v at that latitude
giving c  - v. The GPS data available in (10) also shows that a signal sent westward travels at
speed c plus the rotational speed of the Earth v at that latitude giving c + v."








According to Stokes' rule an integration of angular velocity Ω over an area A is substituted by an integration of tangential component of translational velocity v along the closed line of length L limiting the given area.

Here is the CORIOLIS FORMULA for the MGX:

4AωsinΦ/c2

Φ = (Φ1 + Φ2)/2


Here is the SAGNAC FORMULA for the MGX:

2(V1L1 + V2L2)/c2

V1 = V0cosΦ1
V2 = V0cosΦ2
L1 = L0cosΦ1
L2 = L0cosΦ2

The MGX interferometer, on a rotating Earth, would have been subjected to both effects: CORIOLIS and SAGNAC.

Re: I have questions
« Reply #54 on: October 16, 2019, 04:23:29 PM »
The real reason for the retardation of the counterclock-wise beam can be found here, a paper posted earlier:

http://cdn.intechopen.com/pdfs/39778/InTech-Gps_and_the_one_way_speed_of_light.pdf

"Light travels faster westward than eastward relative to the surface of the Earth. Specifically
the one-way measurement of light speed using GPS data in (6) clearly indicates that a signal
sent eastward travels at speed c minus the rotational speed of the Earth v at that latitude
giving c  - v. The GPS data available in (10) also shows that a signal sent westward travels at
speed c plus the rotational speed of the Earth v at that latitude giving c + v."

Nice...we have made some progress here...

The beam traversing the rectangle in a counter-clockwise direction was retarded because MGPX was performed in Northern Hemisphere.

Had it been performed in Southern Hemisphere the beam traversing the rectangle in clockwise direction would have been retarded.

The question is : Why?

How would you answer this question to someone who is encountering this issue for the first time (from the scratch)?

Re: I have questions
« Reply #55 on: October 16, 2019, 05:18:14 PM »
I have answered it already: on a rotating Earth, the delay is caused by the Coriolis effect of the rotational movement itself. On a stationary Earth (either RE/FE), the delay is caused by the rotating ether drift which is latitude dependent.

Re: I have questions
« Reply #56 on: October 16, 2019, 06:29:22 PM »
I have answered it already: on a rotating Earth, the delay is caused by the Coriolis effect of the rotational movement itself. On a stationary Earth (either RE/FE), the delay is caused by the rotating ether drift which is latitude dependent.

Are you kidding me?

In the southern hemisphere our rectangle has to be rotated 180 degrees, now we are facing south, what is going to happen now?

1. Why would now (unlike in the northern hemisphere) the beam traversing the rectangle in clockwise direction on the spherical earth be retarded?

2. Why would now the beam traversing the rectangle in counter-clockwise direction (just like in the northern hemisphere) on the flat earth be retarded?
« Last Edit: October 16, 2019, 06:30:57 PM by cikljamas »

Re: I have questions
« Reply #57 on: October 16, 2019, 06:39:32 PM »
You are aware, of course, of how the Coriolis force acts on objects on a spherical Earth, are you not?

On a flat earth, the sense of rotation of the ether drift reverses once you cross the equator.

You still cannot explain the MGX unless you make use of my formula.

Re: I have questions
« Reply #58 on: October 16, 2019, 06:52:18 PM »
You are aware, of course, of how the Coriolis force acts on objects on a spherical Earth, are you not?

On a flat earth, the sense of rotation of the ether drift reverses once you cross the equator.

You still cannot explain the MGX unless you make use of my formula.

Sandokhan, i am not sure if you are aware that i am right or you maybe think that for some specific reason i misunderstood something, but :

A) if there is (within my two questions) something/anything that you don't understand feel free to ask, maybe you understand MGPX procedure better than me (i am not afraid to be wrong, are you? ...although i don't think i am wrong in this case, however, i can't be sure 100 % about anything, since we all make mistakes, all the time, don't we?)...
B) if there is something/anything in my two questions that you maybe think that i misunderstood (misinterpreted) for some specific reason, care to point us to my precise logical (or scientific (technical)) error...

So, i repeat :

In the southern hemisphere our rectangle has to be rotated 180 degrees, now we are facing south, what is going to happen now?

1. Why would now (unlike in the northern hemisphere) the beam traversing the rectangle in clockwise direction on the spherical earth be retarded?

2. Why would now the beam traversing the rectangle in counter-clockwise direction (just like in the northern hemisphere) on the flat earth be retarded?

Re: I have questions
« Reply #59 on: October 16, 2019, 06:58:53 PM »
Why would you have both the cw and ccw light beams retarded in the southern hemisphere/semiplane, while you maintain that only the ccw light beam would be retarded in the northern hemisphere/semiplane?